If a*b*c=8 then what is minimum value of (2+a)(2+b)(2+c) [closed]











up vote
1
down vote

favorite
2












If $abc=8$ and $a,b,c >0$, then what is minimum possible value of $(2+a)(2+b)(2+c)$?



Edit: I got the answer and have posted it below.










share|cite|improve this question















closed as off-topic by amWhy, José Carlos Santos, Dietrich Burde, Davide Giraudo, Christopher Nov 15 at 14:20


This question appears to be off-topic. The users who voted to close gave this specific reason:


  • "This question is missing context or other details: Please improve the question by providing additional context, which ideally includes your thoughts on the problem and any attempts you have made to solve it. This information helps others identify where you have difficulties and helps them write answers appropriate to your experience level." – amWhy, José Carlos Santos, Dietrich Burde, Davide Giraudo, Christopher

If this question can be reworded to fit the rules in the help center, please edit the question.













  • What have you tried? AM-GM inequality?
    – астон вілла олоф мэллбэрг
    Nov 15 at 10:23










  • @астонвіллаолофмэллбэрг I didn't tried that actually it didn't click me.
    – Harry Potter
    Nov 15 at 10:25










  • I got answer,thanks
    – Harry Potter
    Nov 15 at 10:26






  • 2




    If you have the answer, please write it down below and accept it.
    – астон вілла олоф мэллбэрг
    Nov 15 at 10:26






  • 1




    You must have some bounds otherwise a=b=c=2 is good because the expression is always increasing positive.
    – NoChance
    Nov 15 at 10:26

















up vote
1
down vote

favorite
2












If $abc=8$ and $a,b,c >0$, then what is minimum possible value of $(2+a)(2+b)(2+c)$?



Edit: I got the answer and have posted it below.










share|cite|improve this question















closed as off-topic by amWhy, José Carlos Santos, Dietrich Burde, Davide Giraudo, Christopher Nov 15 at 14:20


This question appears to be off-topic. The users who voted to close gave this specific reason:


  • "This question is missing context or other details: Please improve the question by providing additional context, which ideally includes your thoughts on the problem and any attempts you have made to solve it. This information helps others identify where you have difficulties and helps them write answers appropriate to your experience level." – amWhy, José Carlos Santos, Dietrich Burde, Davide Giraudo, Christopher

If this question can be reworded to fit the rules in the help center, please edit the question.













  • What have you tried? AM-GM inequality?
    – астон вілла олоф мэллбэрг
    Nov 15 at 10:23










  • @астонвіллаолофмэллбэрг I didn't tried that actually it didn't click me.
    – Harry Potter
    Nov 15 at 10:25










  • I got answer,thanks
    – Harry Potter
    Nov 15 at 10:26






  • 2




    If you have the answer, please write it down below and accept it.
    – астон вілла олоф мэллбэрг
    Nov 15 at 10:26






  • 1




    You must have some bounds otherwise a=b=c=2 is good because the expression is always increasing positive.
    – NoChance
    Nov 15 at 10:26















up vote
1
down vote

favorite
2









up vote
1
down vote

favorite
2






2





If $abc=8$ and $a,b,c >0$, then what is minimum possible value of $(2+a)(2+b)(2+c)$?



Edit: I got the answer and have posted it below.










share|cite|improve this question















If $abc=8$ and $a,b,c >0$, then what is minimum possible value of $(2+a)(2+b)(2+c)$?



Edit: I got the answer and have posted it below.







inequality optimization






share|cite|improve this question















share|cite|improve this question













share|cite|improve this question




share|cite|improve this question








edited Nov 16 at 12:32









user21820

38k541150




38k541150










asked Nov 15 at 10:21









Harry Potter

497




497




closed as off-topic by amWhy, José Carlos Santos, Dietrich Burde, Davide Giraudo, Christopher Nov 15 at 14:20


This question appears to be off-topic. The users who voted to close gave this specific reason:


  • "This question is missing context or other details: Please improve the question by providing additional context, which ideally includes your thoughts on the problem and any attempts you have made to solve it. This information helps others identify where you have difficulties and helps them write answers appropriate to your experience level." – amWhy, José Carlos Santos, Dietrich Burde, Davide Giraudo, Christopher

If this question can be reworded to fit the rules in the help center, please edit the question.




closed as off-topic by amWhy, José Carlos Santos, Dietrich Burde, Davide Giraudo, Christopher Nov 15 at 14:20


This question appears to be off-topic. The users who voted to close gave this specific reason:


  • "This question is missing context or other details: Please improve the question by providing additional context, which ideally includes your thoughts on the problem and any attempts you have made to solve it. This information helps others identify where you have difficulties and helps them write answers appropriate to your experience level." – amWhy, José Carlos Santos, Dietrich Burde, Davide Giraudo, Christopher

If this question can be reworded to fit the rules in the help center, please edit the question.












  • What have you tried? AM-GM inequality?
    – астон вілла олоф мэллбэрг
    Nov 15 at 10:23










  • @астонвіллаолофмэллбэрг I didn't tried that actually it didn't click me.
    – Harry Potter
    Nov 15 at 10:25










  • I got answer,thanks
    – Harry Potter
    Nov 15 at 10:26






  • 2




    If you have the answer, please write it down below and accept it.
    – астон вілла олоф мэллбэрг
    Nov 15 at 10:26






  • 1




    You must have some bounds otherwise a=b=c=2 is good because the expression is always increasing positive.
    – NoChance
    Nov 15 at 10:26




















  • What have you tried? AM-GM inequality?
    – астон вілла олоф мэллбэрг
    Nov 15 at 10:23










  • @астонвіллаолофмэллбэрг I didn't tried that actually it didn't click me.
    – Harry Potter
    Nov 15 at 10:25










  • I got answer,thanks
    – Harry Potter
    Nov 15 at 10:26






  • 2




    If you have the answer, please write it down below and accept it.
    – астон вілла олоф мэллбэрг
    Nov 15 at 10:26






  • 1




    You must have some bounds otherwise a=b=c=2 is good because the expression is always increasing positive.
    – NoChance
    Nov 15 at 10:26


















What have you tried? AM-GM inequality?
– астон вілла олоф мэллбэрг
Nov 15 at 10:23




What have you tried? AM-GM inequality?
– астон вілла олоф мэллбэрг
Nov 15 at 10:23












@астонвіллаолофмэллбэрг I didn't tried that actually it didn't click me.
– Harry Potter
Nov 15 at 10:25




@астонвіллаолофмэллбэрг I didn't tried that actually it didn't click me.
– Harry Potter
Nov 15 at 10:25












I got answer,thanks
– Harry Potter
Nov 15 at 10:26




I got answer,thanks
– Harry Potter
Nov 15 at 10:26




2




2




If you have the answer, please write it down below and accept it.
– астон вілла олоф мэллбэрг
Nov 15 at 10:26




If you have the answer, please write it down below and accept it.
– астон вілла олоф мэллбэрг
Nov 15 at 10:26




1




1




You must have some bounds otherwise a=b=c=2 is good because the expression is always increasing positive.
– NoChance
Nov 15 at 10:26






You must have some bounds otherwise a=b=c=2 is good because the expression is always increasing positive.
– NoChance
Nov 15 at 10:26












2 Answers
2






active

oldest

votes

















up vote
3
down vote













I saw comment of A.M., G.M. inequality and solved it on my own. just posting answerenter image description here






share|cite|improve this answer

















  • 3




    Can you render it into text? Thanks.
    – Oscar Lanzi
    Nov 15 at 10:50










  • How could this lead to values for a,b and c?
    – NoChance
    Nov 15 at 10:52










  • You still have to show in your thesis that number $64$ is the minimum, i.e there exist $a,b,c$ such that....
    – dmtri
    Nov 15 at 17:34


















up vote
2
down vote













You can solve this directly using the method of Lagrange multipliers: the critical points are solutions to
begin{align*}
(2+a)(2+b) + lambda a b &=0\
(2+a)(2+c) + lambda a c &=0\
(2+b)(2+c) + lambda b c &=0\
abc &= 8,
end{align*}

and eliminating equations gives you $a=b=c=2$.



You don't need to consider the case where any of your inequality constraints are active, since your objective function diverges as, say, $ato0$.






share|cite|improve this answer




























    2 Answers
    2






    active

    oldest

    votes








    2 Answers
    2






    active

    oldest

    votes









    active

    oldest

    votes






    active

    oldest

    votes








    up vote
    3
    down vote













    I saw comment of A.M., G.M. inequality and solved it on my own. just posting answerenter image description here






    share|cite|improve this answer

















    • 3




      Can you render it into text? Thanks.
      – Oscar Lanzi
      Nov 15 at 10:50










    • How could this lead to values for a,b and c?
      – NoChance
      Nov 15 at 10:52










    • You still have to show in your thesis that number $64$ is the minimum, i.e there exist $a,b,c$ such that....
      – dmtri
      Nov 15 at 17:34















    up vote
    3
    down vote













    I saw comment of A.M., G.M. inequality and solved it on my own. just posting answerenter image description here






    share|cite|improve this answer

















    • 3




      Can you render it into text? Thanks.
      – Oscar Lanzi
      Nov 15 at 10:50










    • How could this lead to values for a,b and c?
      – NoChance
      Nov 15 at 10:52










    • You still have to show in your thesis that number $64$ is the minimum, i.e there exist $a,b,c$ such that....
      – dmtri
      Nov 15 at 17:34













    up vote
    3
    down vote










    up vote
    3
    down vote









    I saw comment of A.M., G.M. inequality and solved it on my own. just posting answerenter image description here






    share|cite|improve this answer












    I saw comment of A.M., G.M. inequality and solved it on my own. just posting answerenter image description here







    share|cite|improve this answer












    share|cite|improve this answer



    share|cite|improve this answer










    answered Nov 15 at 10:46









    Harry Potter

    497




    497








    • 3




      Can you render it into text? Thanks.
      – Oscar Lanzi
      Nov 15 at 10:50










    • How could this lead to values for a,b and c?
      – NoChance
      Nov 15 at 10:52










    • You still have to show in your thesis that number $64$ is the minimum, i.e there exist $a,b,c$ such that....
      – dmtri
      Nov 15 at 17:34














    • 3




      Can you render it into text? Thanks.
      – Oscar Lanzi
      Nov 15 at 10:50










    • How could this lead to values for a,b and c?
      – NoChance
      Nov 15 at 10:52










    • You still have to show in your thesis that number $64$ is the minimum, i.e there exist $a,b,c$ such that....
      – dmtri
      Nov 15 at 17:34








    3




    3




    Can you render it into text? Thanks.
    – Oscar Lanzi
    Nov 15 at 10:50




    Can you render it into text? Thanks.
    – Oscar Lanzi
    Nov 15 at 10:50












    How could this lead to values for a,b and c?
    – NoChance
    Nov 15 at 10:52




    How could this lead to values for a,b and c?
    – NoChance
    Nov 15 at 10:52












    You still have to show in your thesis that number $64$ is the minimum, i.e there exist $a,b,c$ such that....
    – dmtri
    Nov 15 at 17:34




    You still have to show in your thesis that number $64$ is the minimum, i.e there exist $a,b,c$ such that....
    – dmtri
    Nov 15 at 17:34










    up vote
    2
    down vote













    You can solve this directly using the method of Lagrange multipliers: the critical points are solutions to
    begin{align*}
    (2+a)(2+b) + lambda a b &=0\
    (2+a)(2+c) + lambda a c &=0\
    (2+b)(2+c) + lambda b c &=0\
    abc &= 8,
    end{align*}

    and eliminating equations gives you $a=b=c=2$.



    You don't need to consider the case where any of your inequality constraints are active, since your objective function diverges as, say, $ato0$.






    share|cite|improve this answer

























      up vote
      2
      down vote













      You can solve this directly using the method of Lagrange multipliers: the critical points are solutions to
      begin{align*}
      (2+a)(2+b) + lambda a b &=0\
      (2+a)(2+c) + lambda a c &=0\
      (2+b)(2+c) + lambda b c &=0\
      abc &= 8,
      end{align*}

      and eliminating equations gives you $a=b=c=2$.



      You don't need to consider the case where any of your inequality constraints are active, since your objective function diverges as, say, $ato0$.






      share|cite|improve this answer























        up vote
        2
        down vote










        up vote
        2
        down vote









        You can solve this directly using the method of Lagrange multipliers: the critical points are solutions to
        begin{align*}
        (2+a)(2+b) + lambda a b &=0\
        (2+a)(2+c) + lambda a c &=0\
        (2+b)(2+c) + lambda b c &=0\
        abc &= 8,
        end{align*}

        and eliminating equations gives you $a=b=c=2$.



        You don't need to consider the case where any of your inequality constraints are active, since your objective function diverges as, say, $ato0$.






        share|cite|improve this answer












        You can solve this directly using the method of Lagrange multipliers: the critical points are solutions to
        begin{align*}
        (2+a)(2+b) + lambda a b &=0\
        (2+a)(2+c) + lambda a c &=0\
        (2+b)(2+c) + lambda b c &=0\
        abc &= 8,
        end{align*}

        and eliminating equations gives you $a=b=c=2$.



        You don't need to consider the case where any of your inequality constraints are active, since your objective function diverges as, say, $ato0$.







        share|cite|improve this answer












        share|cite|improve this answer



        share|cite|improve this answer










        answered Nov 15 at 10:42









        user7530

        34.3k759112




        34.3k759112















            Popular posts from this blog

            Plaza Victoria

            In PowerPoint, is there a keyboard shortcut for bulleted / numbered list?

            How to put 3 figures in Latex with 2 figures side by side and 1 below these side by side images but in...